2017 AMC 12A Problems/Problem 9

Problem

Let $S$ be the set of points $(x,y)$ in the coordinate plane such that two of the three quantities $3$, $x+2$, and $y-4$ are equal and the third of the three quantities is no greater than the common value. Which of the following is a correct description of $S$?

$\textbf{(A)}\ \text{a single point} \qquad\textbf{(B)}\ \text{two intersecting lines} \\ \qquad\textbf{(C)}\ \text{three lines whose pairwise intersections are three distinct points} \\ \qquad\textbf{(D)}\ \text{a triangle}\qquad\textbf{(E)}\ \text{three rays with a common point}$

Solution

If the two equal values are $3$ and $x+2$, then $x=1$. Also, $y-4<3$ because 3 is the common value. Solving for $y$, we get $y<7$. Therefore the portion of the line $x=1$ where $y<7$ is part of S. This is a ray with an endpoint of $(1, 7)$

Similar to the process above, we assume that the two equal values are $3$ and $y-4$. Solving the equation $3=y-4$ then $y=7$. Also, $x+2<3$ because 3 is the common value. Solving for $x$, we get $x<1$. Therefore the portion of the line $y=7$ where $x<1$ is also part of S. This is another ray with the same endpoint as the above ray: $(1, 7)$.

If $x+2$ and $y-4$ are the two equal values, then $x+2=y-4$. Solving the equation for $y$, we get $y=x+6$. Also $3<y-4$ because $y-4$ is one way to express the common value. Solving for $y$, we get $y>7$ Therefore the portion of the line $y=x+6$ where $y>7$ is part of S like the other two rays. The lowest possible value that can be achieved is also $(1, 7)$, which makes the answer E.

See Also

2017 AMC 12A (ProblemsAnswer KeyResources)
Preceded by
Problem 8
Followed by
Problem 10
1 2 3 4 5 6 7 8 9 10 11 12 13 14 15 16 17 18 19 20 21 22 23 24 25
All AMC 12 Problems and Solutions

The problems on this page are copyrighted by the Mathematical Association of America's American Mathematics Competitions. AMC logo.png